0
$\begingroup$

One of my interest is 3D shape analysis, and there's a relatively recent framework called functional maps. In the framework of functional maps for shape analysis we represent shape features as a function and such function (assuming Hilbert Space framework) can be expanded onto a base.

If we have for example two shapes (Riemannian Manifolds in general) then transferring features can be seen as mapping coefficients from one base to another. A common way to extract such base is by using the Laplace Beltrami Operator.

A common problem in this framework is to find what is informally as compact base. Suppose a function $f$ and a orthonormal basis $u_1,u_2,\ldots$ is also given then we can represent $f$ as

$$ f = \sum_{i=1}^{+\infty} \alpha_i u_i. $$

We can say that this base is compact if there's a small natural $N$ such that

$$ \left\lVert f \right\rVert^2 \approx \sum_{i=1}^N | \alpha_i |^2 $$

This is suppose to mean that most of the information to represent $f$ can be represented using only the first $N$ basis functions.

I wonder if there's something in the math literature in extracting bases with such property. Or maybe some keywords I could use to find the answer myself (I don't think compact bases is an official term, and it hasn't led me to anything anyway).

$\endgroup$
4
  • $\begingroup$ This question needs more specifics. For instance, I do not think you would consider the choice $u_1=f/\|f\|$ a solution to your problem. $\endgroup$ Commented Jul 4 at 17:51
  • 1
    $\begingroup$ I second Michael's point. You probably mean that your second formula holds - that is, I guess, there are $N\in\mathbb{N}$ and constants $0<c<C$ such that $$c\|f\|^2\leq\sum^N_{i=1}|\alpha_i|^2\leq C\|f\|^2$$ - for all $f$ in a certain (pre?-)Hilbert function space $F$ over a Riemannian manifold $(M,g)$ (maybe $F=L^2(M,g)$ or its subspace of smooth elements?) such that $S=\{u_i\ |\ i\in\mathbb{N}\}\subset F$ is an orthonormal set that is also a topological basis of $F$, so that $(\alpha_i)_{i\in\mathbb{N}}$ is the sequence of components of $f$ in $S$. $\endgroup$ Commented Jul 4 at 18:13
  • $\begingroup$ $N$ should be independent from $f$, just FYI. As mentioned anyway it's some literature reference on the subject, if there's any, that I am looking for. $\endgroup$ Commented Jul 10 at 18:51
  • $\begingroup$ @user8469759 in that case, the situation is as described in the first and last paragraphs of my answer. That also probably explains why you haven't found anything about it in the literature... $\endgroup$ Commented Jul 16 at 22:20

1 Answer 1

0
$\begingroup$

I suppose the missing elements of the setup of your question are as I inferred in my comment to the OP. If so, then any orthonormal basis will have your desired property for a sufficiently large $N\in\mathbb{N}$ depending on $f$. If you want $N$ to be independent of $f$ (which I assume from now on to be what the OP means by a "compact" orthonormal basis) this is only possible if the function space in question is finite dimensional, in which case once more every orthonormal basis is compact. Unfortunately, that makes the concept of a compact orthonormal basis trivial in finite dimensions and void in infinite dimensions. I still believe, though, that the question has pedagogical value (so I upvoted it) because it illustrates an inherent difficulty in making finite dimensional approximations in an infinite dimensional function space which may be relevant in practical applications.

To reach that conclusion, it is more useful to formulate this problem abstractly: let $F$ be a (real or complex) pre-Hilbert space whose scalar product is denoted by $\langle\cdot,\cdot\rangle$ (which we assume to be antilinear in the first variable and linear in the second if $F$ is complex), whose associated norm is denoted by $\|f\|=\sqrt{\langle f,f\rangle}$, $f\in F$. Let $S=\{u_i\ |\ i\in\mathbb{N}\}$ be an orthonormal basis of $F$, that is, $S$ is an orthonormal set in $F$ (i.e. $\langle u_i,u_j\rangle=1$ if $i=j$ and $0$ if $i\neq j$) that is also a topological basis of $F$, that is, for any $f\in F$ there is a unique sequence $(\alpha_i)_{i\in\mathbb{N}}$ of (real or complex) scalars such that $$f=\sum^\infty_{i=1}\alpha_i u_i\ ,$$ with convergence in the (above) norm of $F$. Notice that the countability of $S$ implies that the linear topology of $F$ induced by $\langle\cdot,\cdot\rangle$ is separable. Continuity of the linear functionals $\langle u_i,\cdot\rangle$ then entail that $\alpha_i=\langle u_i,f\rangle$ for all $i\in\mathbb{N}$. Moreover, one then has the Bessel inequality $$\sum_{i\in I}|\langle u_i,f\rangle|^2\leq\|f\|^2$$ for all $I\subset\mathbb{N}$, which allows us to set $C=1$ for all $N\in\mathbb{N}$. Set $P_N f=\sum^N_{i=1}\langle u_i,f\rangle u_i$, $N\in\mathbb{N}$ fixed, so that the sequence $(P_n f)_{n\in\mathbb{N}}$ converges to $f$ in $F$ for all $f\in F$. That convergence for $\frac{1}{\|f\|}f$, $0\neq f\in F$ entails that for all $\epsilon>0$ there is $N_\epsilon\in\mathbb{N}$ such that $$\|f\|^2-\|P_N f\|^2=\|f-P_N f\|^2=\sum^\infty_{i=N+1}|\langle u_i,f\rangle|^2\leq\epsilon^2\|f\|^2$$ for all $N\in\mathbb{N}$ with $N\geq N_\epsilon$. Setting $\epsilon<1$ so that $0<c=1-\epsilon^2<1=C$ yields $$c\|f\|^2\leq\|P_N f\|^2=\sum^N_{i=1}|\langle u_i,f\rangle|^2\leq\|f\|^2$$ (the case $f=0$ is trivially included with equality), where the last inequality is just Bessel's inequality for $I=\{1,\ldots,N\}$.

If you want the stronger property of compactness from the above chosen orthonormal basis $S$, that is, the last chain of inequalities above holding with $N$ independent of $f$, it suffices that this holds for such an $N$ and all $f$ in the unit sphere $K=\{f\in F\ |\ \|f\|=1\}$. If $F$ is finite dimensional, this is automatically true for every orthonormal basis of $F$ since then $K$ is compact. If we asssume that $F$ is infinite dimensional, this amounts to the sequence of finite rank linear operators $(P_n)_{n\in\mathbb{N}}$ converging to the identity in the operator norm topology, which on its turn implies that the identity is a compact linear operator. This implies that the closed balls of $F$ are compact, which can only happen if $F$ is finite dimensional, a contradiction.

$\endgroup$
9
  • $\begingroup$ I understood it differently: $N$ and the basis $\{ u_i \} _{i \ge 1}$ should be independent of $f$, which is the real difficulty of the problem (which otherwise is trivial, as you explain). In your interpretation, every orthonormal basis would be compact (in the OP's definition), which would make the concept useless. $\endgroup$ Commented Jul 5 at 10:41
  • 3
    $\begingroup$ It doesn't really matter: as argued in the last paragraph of my answer, if there is one o.n. basis of $F$ which is compact (in the OP's sense, i.e. with $N$ independent of $f$), then the identity operator of $F$ is the operator norm limit of a sequence of finite rank operators (the orthogonal projections associated to the first $n$ members of the chosen o.n. basis for each $n\in\mathbb{N}$) and hence is a compact operator on $F$. This implies that the closed balls of $F$ are compact, which is only possible if $F$ is finite dimensional, in which case every o.n. basis is compact. $\endgroup$ Commented Jul 5 at 18:05
  • $\begingroup$ I've made the reasoning in the last paragraph clearer in order to emphasize the above point. $\endgroup$ Commented Jul 5 at 18:18
  • $\begingroup$ It is conceivable that there'd be a reformulation of the question, and its context, in which nuclear spaces and/or Schwartz' Kernel Theorem would give an affirmative answer...? $\endgroup$ Commented Jul 5 at 22:14
  • $\begingroup$ One can still make $N$ independent of $f$ belonging to a (relatively) compact subset of $F$, that much is clear. This means that $(P_n)_{n\in\mathbb{N}}$ converges to the identity in the (locally convex) topology of uniform convergence on relatively compact subsets of F, which on its turn is a sequential version of the approximation property, which holds true for any pre-Hilbert space $F$. The latter remains true if $F$ is nuclear because then its topology is induced from a system of Hilbert seminorms. $\endgroup$ Commented Jul 6 at 6:25

You must log in to answer this question.

Start asking to get answers

Find the answer to your question by asking.

Ask question

Explore related questions

See similar questions with these tags.